1 00:00:08,000 --> 00:00:14,000 Well, today is the last day on Laplace transform and the first 2 00:00:12,000 --> 00:00:18,000 day before we start the rest of the term, which will be spent on 3 00:00:16,000 --> 00:00:22,000 the study of systems. I would like to spend it on one 4 00:00:20,000 --> 00:00:26,000 more type of input function which, in general, 5 00:00:23,000 --> 00:00:29,000 your teachers in other courses will expect you to have had some 6 00:00:28,000 --> 00:00:34,000 acquaintance with. It is the kind associated with 7 00:00:32,000 --> 00:00:38,000 an impulse, so an input consisted of what is sometimes 8 00:00:36,000 --> 00:00:42,000 called a unit impulse. Now, what's an impulse? 9 00:00:40,000 --> 00:00:46,000 It covers actually a lot of things. 10 00:00:43,000 --> 00:00:49,000 It covers a situation where you withdraw from a bank account. 11 00:00:48,000 --> 00:00:54,000 For example, take half your money out of a 12 00:00:52,000 --> 00:00:58,000 bank account one day. It also would be modeled the 13 00:00:56,000 --> 00:01:02,000 same way. But the simplest way to 14 00:00:59,000 --> 00:01:05,000 understand it the first time through is as an impulse, 15 00:01:03,000 --> 00:01:09,000 if you know what an impulse is. If you have a variable force 16 00:01:08,000 --> 00:01:14,000 acting over time, and we will assume it is acting 17 00:01:12,000 --> 00:01:18,000 along a straight line so I don't have to worry about it being a 18 00:01:17,000 --> 00:01:23,000 vector, then the impulse, according to physicists, 19 00:01:20,000 --> 00:01:26,000 the physical definition, the impulse of f of t 20 00:01:25,000 --> 00:01:31,000 over some time interval. Let's say the time interval 21 00:01:30,000 --> 00:01:36,000 running from a to b is, by definition, 22 00:01:33,000 --> 00:01:39,000 the integral from a to b of f of t dt. 23 00:01:38,000 --> 00:01:44,000 Actually, I am going to do the 24 00:01:42,000 --> 00:01:48,000 most horrible thing this period. I will assume the force is 25 00:01:46,000 --> 00:01:52,000 actually a constant force. So, in that case, 26 00:01:50,000 --> 00:01:56,000 I wouldn't even have to bother with the integral at all. 27 00:01:55,000 --> 00:02:01,000 If f of t is a constant, let's say capital F, 28 00:01:58,000 --> 00:02:04,000 then the impulse is -- Well, that integral is simply 29 00:02:04,000 --> 00:02:10,000 the product of the two, the impulse over that time 30 00:02:08,000 --> 00:02:14,000 interval is simply F times b minus a. 31 00:02:12,000 --> 00:02:18,000 Just the product of those two. The force times the length of 32 00:02:17,000 --> 00:02:23,000 time for which it acts. Now, that is what I want to 33 00:02:21,000 --> 00:02:27,000 calculate, want to consider in connection with our little mass 34 00:02:26,000 --> 00:02:32,000 system. So, once again, 35 00:02:29,000 --> 00:02:35,000 I think this is probably the last time you'll see the little 36 00:02:34,000 --> 00:02:40,000 spring. Let's bid a tearful farewell to 37 00:02:37,000 --> 00:02:43,000 it. There is our little mass on 38 00:02:39,000 --> 00:02:45,000 wheels. And let's make it an undamped 39 00:02:42,000 --> 00:02:48,000 mass. It has an equilibrium point and 40 00:02:45,000 --> 00:02:51,000 all the other little things that go with the picture. 41 00:02:49,000 --> 00:02:55,000 And when I apply an impulse, what I mean is applying a 42 00:02:54,000 --> 00:03:00,000 constant force to this over a definite time interval. 43 00:03:00,000 --> 00:03:06,000 And that is what I mean by applying an impulse over that 44 00:03:03,000 --> 00:03:09,000 time interval. Now, what is the picture of 45 00:03:06,000 --> 00:03:12,000 such a thing? Well, the force is only going 46 00:03:09,000 --> 00:03:15,000 to be applied, in other words, 47 00:03:11,000 --> 00:03:17,000 I am going to push on the mass or pull on the mass with a 48 00:03:15,000 --> 00:03:21,000 constant force. With a little electromagnet 49 00:03:18,000 --> 00:03:24,000 here, we will assume, there is a pile of iron filings 50 00:03:22,000 --> 00:03:28,000 or something inside there. I turn on the electromagnet. 51 00:03:26,000 --> 00:03:32,000 It pulls with a constant force just between time zero and time 52 00:03:30,000 --> 00:03:36,000 two seconds. And then I stop. 53 00:03:34,000 --> 00:03:40,000 That is going to change the motion of the thing. 54 00:03:37,000 --> 00:03:43,000 First it is going to start pulling it toward the thing. 55 00:03:40,000 --> 00:03:46,000 And then, when it lets go, it will zoom back and there 56 00:03:44,000 --> 00:03:50,000 will be a certain motion after that. 57 00:03:46,000 --> 00:03:52,000 What the question is, if I want to solve that problem 58 00:03:49,000 --> 00:03:55,000 of the motion of that in terms of the Laplace transform, 59 00:03:53,000 --> 00:03:59,000 how am I going to model this force? 60 00:03:55,000 --> 00:04:01,000 Well, let's draw a picture of it first. 61 00:03:59,000 --> 00:04:05,000 It starts here. It is zero for t, 62 00:04:02,000 --> 00:04:08,000 let's say the force is applied between time zero to time h. 63 00:04:08,000 --> 00:04:14,000 And then its force is turned on, it stays constant and then 64 00:04:14,000 --> 00:04:20,000 it is turned off. And those vertical lines 65 00:04:18,000 --> 00:04:24,000 shouldn't be there. But, since in practice, 66 00:04:23,000 --> 00:04:29,000 it takes a tiny bit of time to turn a force on and off. 67 00:04:30,000 --> 00:04:36,000 It is, in practice, not unrealistic to suppose that 68 00:04:34,000 --> 00:04:40,000 there are approximately vertical lines there. 69 00:04:38,000 --> 00:04:44,000 They are slightly slanted but not too much. 70 00:04:41,000 --> 00:04:47,000 Now, I want it to be unit impulse. 71 00:04:44,000 --> 00:04:50,000 This is the force access and this is the time access. 72 00:04:49,000 --> 00:04:55,000 Since the impulse is the area under this curve, 73 00:04:53,000 --> 00:04:59,000 if I want that to be one, then if this is h, 74 00:04:56,000 --> 00:05:02,000 the height to which I -- In other words, 75 00:05:01,000 --> 00:05:07,000 the magnitude of the force must be one over h in order 76 00:05:05,000 --> 00:05:11,000 that the area be one, in order, in other words, 77 00:05:09,000 --> 00:05:15,000 that this integral be one, the area under that curve be 78 00:05:13,000 --> 00:05:19,000 one. So the unit impulse looks like 79 00:05:15,000 --> 00:05:21,000 that. The narrower it is here, 80 00:05:17,000 --> 00:05:23,000 the higher it has to be that way. 81 00:05:20,000 --> 00:05:26,000 The bigger the force must be if you want the end result to be a 82 00:05:24,000 --> 00:05:30,000 unit impulse. Now, to solve a problem, 83 00:05:27,000 --> 00:05:33,000 a typical problem, then, would be a spring. 84 00:05:32,000 --> 00:05:38,000 The mass is traveling on the track. 85 00:05:34,000 --> 00:05:40,000 Let's suppose the spring constant is one, 86 00:05:38,000 --> 00:05:44,000 so there would be a differential equation. 87 00:05:41,000 --> 00:05:47,000 And the right-hand side would be this f of t. 88 00:05:45,000 --> 00:05:51,000 Well, let's give it its name, the name I gave it before. 89 00:05:49,000 --> 00:05:55,000 Remember, I called the unit box function the thing which was one 90 00:05:55,000 --> 00:06:01,000 between zero and h and zero everywhere else. 91 00:06:00,000 --> 00:06:06,000 The notation we used for that was u, and then it had a double 92 00:06:04,000 --> 00:06:10,000 subscript from the starting point and the finishing point. 93 00:06:07,000 --> 00:06:13,000 So oh-- u(oh) of t. 94 00:06:10,000 --> 00:06:16,000 This much represents the thing if it only rose to the high one. 95 00:06:14,000 --> 00:06:20,000 But if it, instead, rises to the height one over h 96 00:06:17,000 --> 00:06:23,000 in order to make that area one, I have to multiply it 97 00:06:21,000 --> 00:06:27,000 by the factor one over h. Now, if you want to solve this 98 00:06:25,000 --> 00:06:31,000 by the Laplace transform. In other words, 99 00:06:28,000 --> 00:06:34,000 see what the motion of that mass is as I apply this unit 100 00:06:32,000 --> 00:06:38,000 impulse to it over that time interval. 101 00:06:34,000 --> 00:06:40,000 You have to take the Laplace transform, if that is the way we 102 00:06:38,000 --> 00:06:44,000 are doing it. Now, the left-hand side is just 103 00:06:41,000 --> 00:06:47,000 routine and would involve the initial conditions. 104 00:06:44,000 --> 00:06:50,000 The whole interest is taking the Laplace transform of the 105 00:06:48,000 --> 00:06:54,000 right-hand side. And that is what I want to do 106 00:06:51,000 --> 00:06:57,000 now. The problem is what is the 107 00:06:52,000 --> 00:06:58,000 Laplace transform of this guy? 108 00:07:02,000 --> 00:07:08,000 Well, remember, to do everything else, 109 00:07:04,000 --> 00:07:10,000 you do everything by writing in terms of the unit step function? 110 00:07:08,000 --> 00:07:14,000 This function that we are talking about is one over h 111 00:07:12,000 --> 00:07:18,000 times what you get by first stepping up to one. 112 00:07:16,000 --> 00:07:22,000 That is the unit step function, which goes up by one and tries 113 00:07:20,000 --> 00:07:26,000 to stay at one ever after. And then, when it gets to h, 114 00:07:24,000 --> 00:07:30,000 it has got to step down. Well, the way you make it step 115 00:07:27,000 --> 00:07:33,000 down is by subtracting off the function, which is the unit step 116 00:07:31,000 --> 00:07:37,000 function but where the step takes place, not at time zero 117 00:07:35,000 --> 00:07:41,000 but at time h. In other words, 118 00:07:38,000 --> 00:07:44,000 I translate the unit step function of course with, 119 00:07:42,000 --> 00:07:48,000 I don't think I have to draw that picture again. 120 00:07:45,000 --> 00:07:51,000 The unit step function looks like zing. 121 00:07:47,000 --> 00:07:53,000 And if you translate it to the right by h it looks like zing. 122 00:07:51,000 --> 00:07:57,000 And then make it negative to subtract it off. 123 00:07:54,000 --> 00:08:00,000 And what you will get is this box function. 124 00:07:56,000 --> 00:08:02,000 So we want to take the Laplace transform of this thing. 125 00:08:01,000 --> 00:08:07,000 Well, let's assume, for the sake of argument that 126 00:08:05,000 --> 00:08:11,000 you didn't remember. Well, you had to use the 127 00:08:08,000 --> 00:08:14,000 formula at 2:00 AM this morning and, therefore, 128 00:08:12,000 --> 00:08:18,000 you do remember it. [LAUGHTER] So I don't have to 129 00:08:16,000 --> 00:08:22,000 recopy the formula onto the board. 130 00:08:19,000 --> 00:08:25,000 Maybe if there is room there. All right, let's put it up 131 00:08:24,000 --> 00:08:30,000 there. It says that u of t minus a 132 00:08:26,000 --> 00:08:32,000 times f, any f, so let's call it g so 133 00:08:31,000 --> 00:08:37,000 you won't confuse it with this particular one, 134 00:08:34,000 --> 00:08:40,000 times g translated. If you translate a function 135 00:08:39,000 --> 00:08:45,000 from t, if you translate it to the right by a then its Laplace 136 00:08:44,000 --> 00:08:50,000 transform is e to the minus a s times whatever the 137 00:08:48,000 --> 00:08:54,000 old Laplace transform was, g of s. 138 00:08:51,000 --> 00:08:57,000 Multiply by an exponential on the right. 139 00:08:54,000 --> 00:09:00,000 On the left that corresponds to translation. 140 00:08:58,000 --> 00:09:04,000 Except you must remember to put in that factor u for a secret 141 00:09:02,000 --> 00:09:08,000 reason which I spent half of Wednesday explaining. 142 00:09:05,000 --> 00:09:11,000 What do we have here? The Laplace transform of u of t, 143 00:09:09,000 --> 00:09:15,000 that is easy. That is simply one over s. 144 00:09:13,000 --> 00:09:19,000 The Laplace transform of this 145 00:09:15,000 --> 00:09:21,000 other guy we get from the formula. 146 00:09:17,000 --> 00:09:23,000 It is basically one over s. No, the Laplace transform of u 147 00:09:21,000 --> 00:09:27,000 of t. But because it has been 148 00:09:24,000 --> 00:09:30,000 translated to the right by h, I have to multiply it by that 149 00:09:28,000 --> 00:09:34,000 factor e to the minus h times s. 150 00:09:38,000 --> 00:09:44,000 That is the answer. And, if you want to solve 151 00:09:40,000 --> 00:09:46,000 problems, this is what you would feed into the equation. 152 00:09:44,000 --> 00:09:50,000 And you would calculate and calculate and calculate it. 153 00:09:47,000 --> 00:09:53,000 But that is not what I want to do now because that was 154 00:09:51,000 --> 00:09:57,000 Wednesday and this is Friday. You have the right to expect 155 00:09:55,000 --> 00:10:01,000 something new. Here is what I am going to do 156 00:09:57,000 --> 00:10:03,000 new. I am going to let h go to zero. 157 00:10:01,000 --> 00:10:07,000 As h goes to zero, this function gets narrower and 158 00:10:06,000 --> 00:10:12,000 narrower, but it also has to get higher and higher because its 159 00:10:12,000 --> 00:10:18,000 area has to stay one. What I am interested in, 160 00:10:16,000 --> 00:10:22,000 first of all, is what happens to the Laplace 161 00:10:20,000 --> 00:10:26,000 transform as h goes to zero. In other words, 162 00:10:25,000 --> 00:10:31,000 what is the limit, as h goes to zero of -- 163 00:10:30,000 --> 00:10:36,000 Well, what is that function? One minus e to the negative hs 164 00:10:34,000 --> 00:10:40,000 divided by hs. 165 00:10:37,000 --> 00:10:43,000 Well, this is an 18.01 problem, an ordinary calculus problem, 166 00:10:42,000 --> 00:10:48,000 but let's do it nicely. You see, the nice way to do it 167 00:10:46,000 --> 00:10:52,000 is to make a substitution. We will change h s to u 168 00:10:50,000 --> 00:10:56,000 because it is occurring as a unit in both cases. 169 00:10:55,000 --> 00:11:01,000 This is going to be the same as the limit as u goes to zero. 170 00:11:00,000 --> 00:11:06,000 I think there are too many u's 171 00:11:04,000 --> 00:11:10,000 here already. I cannot use u, 172 00:11:07,000 --> 00:11:13,000 you cannot use t, v is velocity, 173 00:11:10,000 --> 00:11:16,000 w is wavefunction. There is no letter. 174 00:11:13,000 --> 00:11:19,000 All right, u. It is one minus e to the 175 00:11:17,000 --> 00:11:23,000 negative u over u. 176 00:11:20,000 --> 00:11:26,000 So what is the answer? Well, either you know the 177 00:11:25,000 --> 00:11:31,000 answer or you replace this by, say, the first couple of terms 178 00:11:30,000 --> 00:11:36,000 of the Taylor series. But I think most of you would 179 00:11:35,000 --> 00:11:41,000 use L'Hopital's rule, so let's do that. 180 00:11:38,000 --> 00:11:44,000 The derivative of the top is zero here. 181 00:11:40,000 --> 00:11:46,000 The derivative by the chain rule of e to the negative u is e 182 00:11:44,000 --> 00:11:50,000 to the negative u times minus one. 183 00:11:47,000 --> 00:11:53,000 And that minus one cancels that minus. 184 00:11:49,000 --> 00:11:55,000 So the derivative of the top is simply e to the negative u and 185 00:11:53,000 --> 00:11:59,000 the derivative of the bottom is one. 186 00:11:55,000 --> 00:12:01,000 So, as u goes to zero, that limit is one. 187 00:12:00,000 --> 00:12:06,000 Interesting. Let's draw a picture this way. 188 00:12:03,000 --> 00:12:09,000 I will draw it schematically. Up here is the function one 189 00:12:08,000 --> 00:12:14,000 over h times u zero h of t, our box function, 190 00:12:14,000 --> 00:12:20,000 except it has the height one over h instead of the 191 00:12:19,000 --> 00:12:25,000 height one. We have just calculated that 192 00:12:23,000 --> 00:12:29,000 its Laplace transform is that funny thing, one minus e to the 193 00:12:28,000 --> 00:12:34,000 minus hs divided by hs. 194 00:12:34,000 --> 00:12:40,000 That is the top line. All this is completely kosher, 195 00:12:39,000 --> 00:12:45,000 but now I am going to let h go to zero. 196 00:12:43,000 --> 00:12:49,000 And the question is what do we get now? 197 00:12:47,000 --> 00:12:53,000 Well, I just calculated for you that this thing approaches one, 198 00:12:53,000 --> 00:12:59,000 has the limit one. And now, let's fill in the 199 00:12:58,000 --> 00:13:04,000 picture. What does this thing approach? 200 00:13:03,000 --> 00:13:09,000 Well, it approaches a function which is zero everywhere. 201 00:13:10,000 --> 00:13:16,000 As h approaches zero, this green box turns into a box 202 00:13:17,000 --> 00:13:23,000 which is zero everywhere except at zero. 203 00:13:22,000 --> 00:13:28,000 And there, it is infinitely high. 204 00:13:26,000 --> 00:13:32,000 So, keep going up. 205 00:13:35,000 --> 00:13:41,000 Now, of course, that is not a function. 206 00:13:38,000 --> 00:13:44,000 People call it a function but it isn't. 207 00:13:41,000 --> 00:13:47,000 Mathematicians call it a generalized function, 208 00:13:45,000 --> 00:13:51,000 but that is not a function either. 209 00:13:48,000 --> 00:13:54,000 It is just a way of making you feel comfortable by talking 210 00:13:53,000 --> 00:13:59,000 about something which isn't really a function. 211 00:13:56,000 --> 00:14:02,000 It was given the name, introduced formally into 212 00:14:00,000 --> 00:14:06,000 mathematics by a physicist, Dirac. 213 00:14:05,000 --> 00:14:11,000 And he, looking ahead to the future, did what many people do 214 00:14:09,000 --> 00:14:15,000 who introduce something into the literature, a formula or a 215 00:14:13,000 --> 00:14:19,000 function or something which they think is going to be important. 216 00:14:17,000 --> 00:14:23,000 They never name it directly after themselves, 217 00:14:20,000 --> 00:14:26,000 but they always use as the symbol for it the first letter 218 00:14:24,000 --> 00:14:30,000 of their name. I cannot tell you how often 219 00:14:26,000 --> 00:14:32,000 that has happened. Maybe even Euler called e for 220 00:14:31,000 --> 00:14:37,000 that reason, although he claims it was in Latin because it has 221 00:14:37,000 --> 00:14:43,000 to do with exponentials. Well, luckily his name began 222 00:14:42,000 --> 00:14:48,000 with an E, too. That is Paul Dirac's delta 223 00:14:45,000 --> 00:14:51,000 function. I won't dignify it by the name 224 00:14:49,000 --> 00:14:55,000 function by writing that out, by putting the world function 225 00:14:54,000 --> 00:15:00,000 here, too, but it is called the delta function. 226 00:15:00,000 --> 00:15:06,000 From this point on, the entire rest of the lecture 227 00:15:03,000 --> 00:15:09,000 has a slight fictional element. The entire rest of the lecture 228 00:15:08,000 --> 00:15:14,000 is in figurative quotation marks, so you are not entirely 229 00:15:12,000 --> 00:15:18,000 responsible for anything I say. This is a non-function, 230 00:15:16,000 --> 00:15:22,000 but you put it in there and call it a function. 231 00:15:19,000 --> 00:15:25,000 And you naturally want to complete, if it's a function 232 00:15:23,000 --> 00:15:29,000 then it must have a Laplace transform, even though it 233 00:15:27,000 --> 00:15:33,000 doesn't, so the diagram is completed that way. 234 00:15:32,000 --> 00:15:38,000 And its Laplace transform is declared to be one. 235 00:15:35,000 --> 00:15:41,000 So let's start listing the properties of this weird thing. 236 00:15:54,000 --> 00:16:00,000 The delta function, its Laplace transform is one. 237 00:16:05,000 --> 00:16:11,000 Now, one of the things is we have not yet expressed the fact 238 00:16:09,000 --> 00:16:15,000 that it is a unit impulse. In other words, 239 00:16:13,000 --> 00:16:19,000 since the areas of all of these boxes, they all have areas one 240 00:16:17,000 --> 00:16:23,000 as they are shrunk this way they get higher that way. 241 00:16:22,000 --> 00:16:28,000 By convention, one says that the area under 242 00:16:25,000 --> 00:16:31,000 the orange curve also remains one in the limit. 243 00:16:30,000 --> 00:16:36,000 Now, how am I going to express that? 244 00:16:32,000 --> 00:16:38,000 Well, it is done by the following formula that the 245 00:16:35,000 --> 00:16:41,000 integral, the total impulse of the delta function should be 246 00:16:39,000 --> 00:16:45,000 one. Now, where do I integrate? 247 00:16:41,000 --> 00:16:47,000 Well, from any place that it is zero to any place that it is 248 00:16:45,000 --> 00:16:51,000 zero on the other side of that vertical line. 249 00:16:48,000 --> 00:16:54,000 But, in order to avoid controversy, people integrate 250 00:16:52,000 --> 00:16:58,000 all the way from negative infinity to infinity since it 251 00:16:55,000 --> 00:17:01,000 doesn't hurt. Does it? 252 00:16:57,000 --> 00:17:03,000 It is zero practically all the time. 253 00:17:01,000 --> 00:17:07,000 This is the function whose Laplace transform is one. 254 00:17:06,000 --> 00:17:12,000 Its integral from minus infinity to infinity is one. 255 00:17:11,000 --> 00:17:17,000 How else can we calculate for it? 256 00:17:15,000 --> 00:17:21,000 Well, I would like to calculate its convolution. 257 00:17:20,000 --> 00:17:26,000 Here is f of t. What happens if I convolute it 258 00:17:26,000 --> 00:17:32,000 with the delta function? Well, if you go back to the 259 00:17:31,000 --> 00:17:37,000 definition of the convolution, you know, it is that funny 260 00:17:35,000 --> 00:17:41,000 integral, you are going to do a lot of head scratching because 261 00:17:40,000 --> 00:17:46,000 it is not really all that clear how to integrate with the delta 262 00:17:44,000 --> 00:17:50,000 function. Instead of doing that let's 263 00:17:46,000 --> 00:17:52,000 assume that it follows the laws of the Laplace transform. 264 00:17:50,000 --> 00:17:56,000 In that case, its Laplace transform would be 265 00:17:53,000 --> 00:17:59,000 what? Well, the whole thing of a 266 00:17:55,000 --> 00:18:01,000 convolution is that the Laplace transform of the convolution is 267 00:18:00,000 --> 00:18:06,000 the product of the two separate Laplace transforms. 268 00:18:05,000 --> 00:18:11,000 So that is going to be F of s times the Laplace 269 00:18:09,000 --> 00:18:15,000 transform of the delta function, which is one. 270 00:18:13,000 --> 00:18:19,000 Now, what must this thing be? Well, there is some ambiguity 271 00:18:18,000 --> 00:18:24,000 as to what it is for negative values of t. 272 00:18:21,000 --> 00:18:27,000 But if we, by brute force, decide for negative values of t 273 00:18:26,000 --> 00:18:32,000 it is going to have the value zero, that is the way we make 274 00:18:31,000 --> 00:18:37,000 things unique. In fact, why don't we make f of 275 00:18:35,000 --> 00:18:41,000 unique that way to start with? 276 00:18:38,000 --> 00:18:44,000 This is a function now that is allowed to do anything it wants 277 00:18:41,000 --> 00:18:47,000 on the right-hand side of zero starting at zero, 278 00:18:44,000 --> 00:18:50,000 but on the left-hand side of zero it is wiped away and must 279 00:18:48,000 --> 00:18:54,000 be zero. This is a definite thing now. 280 00:18:50,000 --> 00:18:56,000 Its convolution is this. And the inverse Laplace 281 00:18:53,000 --> 00:18:59,000 transform is -- The answer, in other words, 282 00:18:57,000 --> 00:19:03,000 is the same thing as what u of t f of t would be. 283 00:19:02,000 --> 00:19:08,000 It's the same thing, F of s. 284 00:19:04,000 --> 00:19:10,000 And so, the conclusion is that these are equal, 285 00:19:08,000 --> 00:19:14,000 since they must be unique. They have been made unique by 286 00:19:12,000 --> 00:19:18,000 making them zero for t negative. In other words, 287 00:19:16,000 --> 00:19:22,000 apply to a function, well, I won't recopy it. 288 00:19:19,000 --> 00:19:25,000 But the point is that delta t, for the convolution operation, 289 00:19:24,000 --> 00:19:30,000 is acting like an identity. If I multiply, 290 00:19:29,000 --> 00:19:35,000 in the sense of convolution, it is a peculiar operation. 291 00:19:33,000 --> 00:19:39,000 But algebraically, it has a lot of the properties 292 00:19:36,000 --> 00:19:42,000 of multiplication. It is communitive. 293 00:19:39,000 --> 00:19:45,000 It is linear in both factors. In other words, 294 00:19:42,000 --> 00:19:48,000 it is almost anything you would want with multiplication. 295 00:19:46,000 --> 00:19:52,000 It has an identity element, identity function. 296 00:19:49,000 --> 00:19:55,000 And the identity function is the Dirac delta function. 297 00:19:53,000 --> 00:19:59,000 Anything else here? Yeah, I will throw in one more 298 00:19:57,000 --> 00:20:03,000 thing. It would just require one more 299 00:20:01,000 --> 00:20:07,000 phony argument, which I won't bother giving 300 00:20:04,000 --> 00:20:10,000 you, but it is not totally implausible. 301 00:20:06,000 --> 00:20:12,000 After all, u of t, the unit step function is not 302 00:20:11,000 --> 00:20:17,000 differentiable, is not a differentiable 303 00:20:13,000 --> 00:20:19,000 function. It looks like this. 304 00:20:15,000 --> 00:20:21,000 Here its derivative is zero, here its derivative is zero, 305 00:20:19,000 --> 00:20:25,000 and in this class it is not even defined in between. 306 00:20:23,000 --> 00:20:29,000 But, I don't care, I will make it go straight up. 307 00:20:27,000 --> 00:20:33,000 The question is what's its derivative? 308 00:20:31,000 --> 00:20:37,000 Well, zero here, zero there and infinity at 309 00:20:34,000 --> 00:20:40,000 zero, so it must be the delta function. 310 00:20:37,000 --> 00:20:43,000 That has exactly the right properties. 311 00:20:40,000 --> 00:20:46,000 So the same people who will tell you this will tell you that 312 00:20:44,000 --> 00:20:50,000 also. And, in fact, 313 00:20:45,000 --> 00:20:51,000 when you use it to solve differential equations it acts 314 00:20:50,000 --> 00:20:56,000 as if that is true. I think I have given you an 315 00:20:53,000 --> 00:20:59,000 example on your homework. Let me now show you a typical 316 00:20:57,000 --> 00:21:03,000 example of the way the Dirac delta function would be used to 317 00:21:02,000 --> 00:21:08,000 solve a problem. Let's go back to our little 318 00:21:06,000 --> 00:21:12,000 spring, since it is the easiest thing. 319 00:21:09,000 --> 00:21:15,000 You are familiar with it from a physical point of view, 320 00:21:13,000 --> 00:21:19,000 and it is the easiest thing to illustrate on. 321 00:21:16,000 --> 00:21:22,000 We have our spring mass system. Where is it? 322 00:21:20,000 --> 00:21:26,000 Is it on the board? Up there. 323 00:21:22,000 --> 00:21:28,000 That one. And the differential equation 324 00:21:25,000 --> 00:21:31,000 we are going to solve is y double prime plus y 325 00:21:29,000 --> 00:21:35,000 equals -- And now, I am going to assume 326 00:21:33,000 --> 00:21:39,000 that the spring is kicked with impulse a. 327 00:21:37,000 --> 00:21:43,000 I am not going to kick it at time t equals zero, 328 00:21:42,000 --> 00:21:48,000 since that would get us into slight technical difficulties. 329 00:21:47,000 --> 00:21:53,000 Anyway, it is more fun to kick it at time pi over two. 330 00:21:52,000 --> 00:21:58,000 The thing is, 331 00:21:54,000 --> 00:22:00,000 what is happening? Well, we have got to have 332 00:21:58,000 --> 00:22:04,000 initial conditions. The initial conditions are 333 00:22:02,000 --> 00:22:08,000 going to be, let's start at time zero. 334 00:22:05,000 --> 00:22:11,000 We will start it at the position one. 335 00:22:08,000 --> 00:22:14,000 So I take my spring, I drag it to the position one, 336 00:22:11,000 --> 00:22:17,000 I take the little mass there and then let it go. 337 00:22:15,000 --> 00:22:21,000 And so it starts going birr. But right when it gets to the 338 00:22:19,000 --> 00:22:25,000 equilibrium point I give it a, "cha!" with unit impulse. 339 00:22:23,000 --> 00:22:29,000 I started it from rest. Those will be the initial 340 00:22:27,000 --> 00:22:33,000 conditions. And I want to say that I kicked 341 00:22:31,000 --> 00:22:37,000 it, not with unit impulse, but with the impulse a. 342 00:22:35,000 --> 00:22:41,000 Bigger. And I did that at time pi over 343 00:22:38,000 --> 00:22:44,000 two. So how are we going to say 344 00:22:41,000 --> 00:22:47,000 that? Well, kick it means delivered 345 00:22:43,000 --> 00:22:49,000 that impulse over an extremely short time interval, 346 00:22:47,000 --> 00:22:53,000 but in such a way kicked it sufficiently hard that the total 347 00:22:52,000 --> 00:22:58,000 impulse was a. The way to say that is kick it 348 00:22:56,000 --> 00:23:02,000 with the Dirac delta function. Translate it to the point time 349 00:23:02,000 --> 00:23:08,000 pi over two. Not at zero any longer. 350 00:23:06,000 --> 00:23:12,000 t minus pi over two. 351 00:23:09,000 --> 00:23:15,000 But that would kick it with a unit impulse. 352 00:23:12,000 --> 00:23:18,000 I want it to kick it with the impulse a, so I will just 353 00:23:16,000 --> 00:23:22,000 multiply that by the constant factor a. 354 00:23:20,000 --> 00:23:26,000 Let's put this over here. y of zero equals one, 355 00:23:24,000 --> 00:23:30,000 that's the starting value. Now we have a problem. 356 00:23:30,000 --> 00:23:36,000 The only thing new in solving this with the Laplace transform 357 00:23:33,000 --> 00:23:39,000 is I have this funny right-hand side. 358 00:23:36,000 --> 00:23:42,000 But it corresponds to a physical situation. 359 00:23:38,000 --> 00:23:44,000 Let's do it. You take the Laplace transform 360 00:23:41,000 --> 00:23:47,000 of both sides of the equation. Remember how to do that? 361 00:23:44,000 --> 00:23:50,000 You have to take account of the initial conditions. 362 00:23:48,000 --> 00:23:54,000 The Laplace transform of the second derivative is you 363 00:23:51,000 --> 00:23:57,000 multiply by s squared, and then you have to subtract. 364 00:23:55,000 --> 00:24:01,000 You have to use these initial conditions. 365 00:23:59,000 --> 00:24:05,000 This one won't give you anything, but the first one 366 00:24:02,000 --> 00:24:08,000 means I have to subtract one times s. 367 00:24:05,000 --> 00:24:11,000 That is the Laplace transform of y double prime. 368 00:24:09,000 --> 00:24:15,000 The Laplace transform of y, of course, is just capital Y. 369 00:24:13,000 --> 00:24:19,000 And how about the Laplace 370 00:24:16,000 --> 00:24:22,000 transform of the right-hand side. 371 00:24:18,000 --> 00:24:24,000 Well, we will have the constant factor a because the Laplace 372 00:24:22,000 --> 00:24:28,000 transform is linear. And now, the delta function 373 00:24:25,000 --> 00:24:31,000 would have the transform one. But when I translate it, 374 00:24:30,000 --> 00:24:36,000 pi over two, that means I have to use that 375 00:24:33,000 --> 00:24:39,000 formula. Translate it by pi over two 376 00:24:35,000 --> 00:24:41,000 means take the one that it would have been otherwise and multiply 377 00:24:40,000 --> 00:24:46,000 it by e, that exponential factor. 378 00:24:42,000 --> 00:24:48,000 It would be e to the minus pi over two, 379 00:24:46,000 --> 00:24:52,000 that is the A times s times one, which would be the g of s, 380 00:24:49,000 --> 00:24:55,000 the Laplace transform or the delta function before it 381 00:24:54,000 --> 00:25:00,000 had been translated. But I don't have to put that in 382 00:24:57,000 --> 00:25:03,000 because it's one. I am multiplying by one. 383 00:25:01,000 --> 00:25:07,000 And to do everything now is routine. 384 00:25:03,000 --> 00:25:09,000 Solve for the Laplace transform. 385 00:25:05,000 --> 00:25:11,000 Well, what is it? It is y is equal to. 386 00:25:08,000 --> 00:25:14,000 I put the s on the other side. That makes the right-hand side 387 00:25:12,000 --> 00:25:18,000 the sum of two terms. And I divide by the coefficient 388 00:25:15,000 --> 00:25:21,000 of y, which is s squared plus one. 389 00:25:18,000 --> 00:25:24,000 The s is over on the right-hand side and it is divided by s 390 00:25:22,000 --> 00:25:28,000 squared plus one. And the other factor is there, 391 00:25:25,000 --> 00:25:31,000 too. And it, too, 392 00:25:26,000 --> 00:25:32,000 is divided by s squared plus one. 393 00:25:35,000 --> 00:25:41,000 Now, we take the inverse Laplace transform of those two 394 00:25:38,000 --> 00:25:44,000 terms and add them up. 395 00:26:00,000 --> 00:26:06,000 What will we get? Well, y is equal to, 396 00:26:02,000 --> 00:26:08,000 the inverse Laplace transform of s over s squared plus one is 397 00:26:07,000 --> 00:26:13,000 cosine t. 398 00:26:11,000 --> 00:26:17,000 Now, for this thing we will have to use our formula. 399 00:26:15,000 --> 00:26:21,000 If this weren't here, the inverse Laplace transform 400 00:26:19,000 --> 00:26:25,000 of a over s squared plus one would 401 00:26:24,000 --> 00:26:30,000 be what? Well, it would be a times the 402 00:26:27,000 --> 00:26:33,000 sine of t. 403 00:26:35,000 --> 00:26:41,000 In other words, if this is the g of s 404 00:26:37,000 --> 00:26:43,000 then the function on the left would be basically A sine t. 405 00:26:41,000 --> 00:26:47,000 But because it has been 406 00:26:43,000 --> 00:26:49,000 multiplied by that exponential factor, e to the minus as 407 00:26:47,000 --> 00:26:53,000 where a is pi over two, 408 00:26:50,000 --> 00:26:56,000 the left-hand side has to be changed from A sine t 409 00:26:53,000 --> 00:26:59,000 to what it would be with the translated form. 410 00:26:58,000 --> 00:27:04,000 So the rest of it is u of t minus pi over two, 411 00:27:01,000 --> 00:27:07,000 because a is pi over two, times what it would have 412 00:27:05,000 --> 00:27:11,000 been just from the factor g of s itself. 413 00:27:09,000 --> 00:27:15,000 In other words, A times the sine of, 414 00:27:11,000 --> 00:27:17,000 again, t minus pi over two. 415 00:27:15,000 --> 00:27:21,000 I am applying that formula, but I am applying it in that 416 00:27:19,000 --> 00:27:25,000 direction. I started with this, 417 00:27:21,000 --> 00:27:27,000 and I want to recover the left-hand side. 418 00:27:24,000 --> 00:27:30,000 And that is what it must look like. 419 00:27:26,000 --> 00:27:32,000 The A, of course, just gets dragged along for the 420 00:27:29,000 --> 00:27:35,000 free ride. Now, as I emphasized to you 421 00:27:34,000 --> 00:27:40,000 last time, and I hope you did on your homework that you handed 422 00:27:38,000 --> 00:27:44,000 in, you mustn't leave it in that form. 423 00:27:41,000 --> 00:27:47,000 You have to make cases because people will expect you to tell 424 00:27:46,000 --> 00:27:52,000 them what the meaning of this is. 425 00:27:49,000 --> 00:27:55,000 Now, if t is less than pi over two, this is zero. 426 00:27:52,000 --> 00:27:58,000 And, therefore, that term does not exist. 427 00:27:56,000 --> 00:28:02,000 So the first part of it is just the cosine t term if 428 00:28:00,000 --> 00:28:06,000 t lies between zero and pi over two. 429 00:28:05,000 --> 00:28:11,000 If t is bigger than pi over two then this factor is 430 00:28:10,000 --> 00:28:16,000 one. It's the unit step function. 431 00:28:12,000 --> 00:28:18,000 And I, therefore, must add in this term. 432 00:28:16,000 --> 00:28:22,000 Now, what is that term? What is the sine of t minus pi 433 00:28:20,000 --> 00:28:26,000 over two? The sine of t looks 434 00:28:25,000 --> 00:28:31,000 like that. The sine of t, 435 00:28:27,000 --> 00:28:33,000 if I translate it, looks like this. 436 00:28:31,000 --> 00:28:37,000 If I translate it by pi over two. 437 00:28:33,000 --> 00:28:39,000 And let's finish it up, the pi that was over here moved 438 00:28:38,000 --> 00:28:44,000 into position. That curve is the curve 439 00:28:41,000 --> 00:28:47,000 negative cosine t. 440 00:28:51,000 --> 00:28:57,000 And so the answer is if t is bigger than pi over two, 441 00:28:55,000 --> 00:29:01,000 it is cosine t minus A times cosine t. 442 00:29:00,000 --> 00:29:06,000 Or, in other words, 443 00:29:02,000 --> 00:29:08,000 it is one minus A times cosine t. 444 00:29:11,000 --> 00:29:17,000 Now, do those match up? They have always got to match 445 00:29:13,000 --> 00:29:19,000 up, or you have made a mistake. You always have to get a 446 00:29:17,000 --> 00:29:23,000 continuous function when you have just discontinuities. 447 00:29:20,000 --> 00:29:26,000 Do we get a continuous function? 448 00:29:22,000 --> 00:29:28,000 Yeah, when t is pi over two the value here is 449 00:29:25,000 --> 00:29:31,000 zero. The value of this is also zero 450 00:29:27,000 --> 00:29:33,000 at pi over two. There is no conflict in the 451 00:29:31,000 --> 00:29:37,000 values. Values doesn't suddenly jump. 452 00:29:33,000 --> 00:29:39,000 The function is continuous. It is not differential but it 453 00:29:38,000 --> 00:29:44,000 is continuous. Well, what function does that 454 00:29:41,000 --> 00:29:47,000 look like? There are cases. 455 00:29:43,000 --> 00:29:49,000 It starts out life as the function cosine t. 456 00:29:47,000 --> 00:29:53,000 So it gets to here. And at t equals pi over two, 457 00:29:51,000 --> 00:29:57,000 the mass gets kicked and that changes the function. 458 00:29:55,000 --> 00:30:01,000 Now, what are the values? Well, if A is bigger than one 459 00:30:01,000 --> 00:30:07,000 this is a negative number and it therefore becomes 460 00:30:07,000 --> 00:30:13,000 the function negative cosine t. 461 00:30:11,000 --> 00:30:17,000 Now, negative cosine t looks like this, the blue guy. 462 00:30:16,000 --> 00:30:22,000 Negative cosine t is a function that looks like this. 463 00:30:21,000 --> 00:30:27,000 So it goes from here, it reverses direction, 464 00:30:26,000 --> 00:30:32,000 the mass reverses direction from what you thought it was 465 00:30:31,000 --> 00:30:37,000 going to do. And it does that because A is 466 00:30:36,000 --> 00:30:42,000 so large that that impulse was enough to make it reverse 467 00:30:40,000 --> 00:30:46,000 direction. Of course it might only do 468 00:30:42,000 --> 00:30:48,000 this, but this is what will happen if A is bigger than one. 469 00:30:47,000 --> 00:30:53,000 This will be A, which is a lot bigger than one. 470 00:30:50,000 --> 00:30:56,000 If it's not so much bigger than one it might look like that. 471 00:30:55,000 --> 00:31:01,000 So A is just bigger than one. How's that? 472 00:30:59,000 --> 00:31:05,000 Well, what if A is less than one? 473 00:31:01,000 --> 00:31:07,000 Well, in that case it stays positive. 474 00:31:04,000 --> 00:31:10,000 If A is less than one, this is now still a positive 475 00:31:07,000 --> 00:31:13,000 number. And, therefore, 476 00:31:09,000 --> 00:31:15,000 the cosine continues on its merry way. 477 00:31:12,000 --> 00:31:18,000 The only thing is it might be a little more sluggish or it might 478 00:31:17,000 --> 00:31:23,000 be very peppy and do that. Let's just go that far. 479 00:31:20,000 --> 00:31:26,000 This will be the case A less than one. 480 00:31:23,000 --> 00:31:29,000 Well, of course, the most interesting case is 481 00:31:26,000 --> 00:31:32,000 what happens if A is exactly equal to one? 482 00:31:32,000 --> 00:31:38,000 The porridge is exactly just right, I think that's the 483 00:31:37,000 --> 00:31:43,000 phrase. Too hot. 484 00:31:38,000 --> 00:31:44,000 Too cold. Just right. 485 00:31:40,000 --> 00:31:46,000 When A is equal to one, it is zero. 486 00:31:44,000 --> 00:31:50,000 It starts out as cosine t. 487 00:31:48,000 --> 00:31:54,000 When it gets to t, it continues on ever after as 488 00:31:52,000 --> 00:31:58,000 the function zero. I have a visual aid for the 489 00:31:57,000 --> 00:32:03,000 only time this term. It didn't work at all. 490 00:32:02,000 --> 00:32:08,000 I mean, on the other hand, the last hour, 491 00:32:06,000 --> 00:32:12,000 the people who worked it were not intrinsically baseball 492 00:32:11,000 --> 00:32:17,000 players, so we will use the equation of the pendulum 493 00:32:15,000 --> 00:32:21,000 instead. That is a lot easier than mass 494 00:32:18,000 --> 00:32:24,000 spring. This is a pendulum. 495 00:32:20,000 --> 00:32:26,000 It is undamped because I declare it to be and it swings 496 00:32:25,000 --> 00:32:31,000 back and forth. And here I am releasing it. 497 00:32:30,000 --> 00:32:36,000 The variable is not x or y but theta, the angle through. 498 00:32:34,000 --> 00:32:40,000 Here theta is one, let's say. 499 00:32:37,000 --> 00:32:43,000 That's about one radian. It starts there and swings back 500 00:32:41,000 --> 00:32:47,000 and forth. It is not damped, 501 00:32:44,000 --> 00:32:50,000 so it never loses amplitude, particularly if I swish it, 502 00:32:48,000 --> 00:32:54,000 if I move my hand a little bit. I want someone who knows how to 503 00:32:53,000 --> 00:32:59,000 bat a baseball. That was the problem last hour. 504 00:32:57,000 --> 00:33:03,000 Two people. One to release it. 505 00:33:01,000 --> 00:33:07,000 I will stand up and try to hold it here. 506 00:33:04,000 --> 00:33:10,000 Somebody releases it. And then somebody who has to be 507 00:33:08,000 --> 00:33:14,000 very skillful should apply a unit impulse of exactly one when 508 00:33:13,000 --> 00:33:19,000 it gets to the equilibrium point. 509 00:33:16,000 --> 00:33:22,000 So who can do that? Who can play baseball here? 510 00:33:20,000 --> 00:33:26,000 Come on. Somebody elected? 511 00:33:30,000 --> 00:33:36,000 All right. Come on. [APPLAUSE] 512 00:33:41,000 --> 00:33:47,000 Somebody release it, too. 513 00:33:44,000 --> 00:33:50,000 Somebody tall to handle it all. I think that will be me. 514 00:33:52,000 --> 00:33:58,000 Just hold it at what you would take to be one radian. 515 00:34:00,000 --> 00:34:06,000 He releases it. When it gets to the bottom, 516 00:34:04,000 --> 00:34:10,000 you will have to get way down, and maybe on this side. 517 00:34:11,000 --> 00:34:17,000 Are you a lefty or a righty? Rightly. 518 00:34:15,000 --> 00:34:21,000 Okay. Bat it what part. 519 00:34:17,000 --> 00:34:23,000 Give it a good swat. I will stand up higher. 520 00:34:22,000 --> 00:34:28,000 Help. I'm not very stable. 521 00:34:25,000 --> 00:34:31,000 [APPLAUSE] A trial run. Again. 522 00:34:30,000 --> 00:34:36,000 Okay. A little further out. 523 00:34:32,000 --> 00:34:38,000 First of all, you have to see where it's 524 00:34:36,000 --> 00:34:42,000 going. Why don't you stand, 525 00:34:38,000 --> 00:34:44,000 oh, you bat rightly. That's right. 526 00:34:41,000 --> 00:34:47,000 Okay. Let's try it again. 527 00:34:59,000 --> 00:35:05,000 Strike one. It's okay. It's the beginning of the baseball season. One more. The Red Sox are having trouble, too. Not bad. [APPLAUSE] 528 00:35:13,000 --> 00:35:19,000 If he had hit even harder it would have reversed direction 529 00:35:16,000 --> 00:35:22,000 and gone that way. If you hadn't hit it quite as 530 00:35:20,000 --> 00:35:26,000 hard it would have continued on, still at cosine t, 531 00:35:24,000 --> 00:35:30,000 but with less amplitude. But if you hit it exactly right 532 00:35:28,000 --> 00:35:34,000 -- It is fun to try to do. 533 00:35:31,000 --> 00:35:37,000 Toomre in our department is a master at this, 534 00:35:36,000 --> 00:35:42,000 but he has been practicing for years. 535 00:35:40,000 --> 00:35:46,000 He can take a little mallet and go blunk, and it stops 536 00:35:45,000 --> 00:35:51,000 absolutely dead. It is unbelievable. 537 00:35:49,000 --> 00:35:55,000 I should have had him give the lecture. 538 00:35:53,000 --> 00:35:59,000 Now, I would like to do something truly serious. 539 00:36:00,000 --> 00:36:06,000 Here, I guess. Because there is a certain 540 00:36:03,000 --> 00:36:09,000 amount of engineering lingo you have to learn. 541 00:36:07,000 --> 00:36:13,000 It is used by almost everybody. Not architects and biologists 542 00:36:12,000 --> 00:36:18,000 probably quite yet, but anybody that uses the 543 00:36:16,000 --> 00:36:22,000 Laplace transform will use these words in connection with it. 544 00:36:21,000 --> 00:36:27,000 I really think, since it is such a widespread 545 00:36:25,000 --> 00:36:31,000 technique, that these are things you should know. 546 00:36:31,000 --> 00:36:37,000 Anyway, it will be easy. It is just the enrichment of 547 00:36:34,000 --> 00:36:40,000 your vocabulary. It is always fun to learn new 548 00:36:37,000 --> 00:36:43,000 vocabulary words. So, let's just consider a 549 00:36:40,000 --> 00:36:46,000 general second order equation. By the way, all this applies to 550 00:36:45,000 --> 00:36:51,000 higher order equations, too. 551 00:36:47,000 --> 00:36:53,000 It applies to systems. The same words are used, 552 00:36:50,000 --> 00:36:56,000 but let's use something that you know. 553 00:36:52,000 --> 00:36:58,000 Here is a system. It could be a spring mass 554 00:36:55,000 --> 00:37:01,000 dashpot system. It could be an RLC circuit. 555 00:37:00,000 --> 00:37:06,000 Or that pendulum, a damped pendulum, 556 00:37:02,000 --> 00:37:08,000 anything that is modeled by that differential equation with 557 00:37:06,000 --> 00:37:12,000 constant coefficients, second-order. 558 00:37:08,000 --> 00:37:14,000 This is the input. The input can be any kind of a 559 00:37:11,000 --> 00:37:17,000 function. Exponential functions, 560 00:37:13,000 --> 00:37:19,000 sine, cosine. It could be a Dirac delta 561 00:37:16,000 --> 00:37:22,000 function. It could be a sum of these 562 00:37:18,000 --> 00:37:24,000 things. It could be a Fourier series. 563 00:37:21,000 --> 00:37:27,000 Anything of the sort of stuff we have been talking about 564 00:37:24,000 --> 00:37:30,000 throughout the last few weeks. And let's have simple initial 565 00:37:30,000 --> 00:37:36,000 conditions so that doesn't louse things up, the simplest possible 566 00:37:34,000 --> 00:37:40,000 ones. The mass starts at the 567 00:37:36,000 --> 00:37:42,000 equilibrium point from rest. Of course, it doesn't stay that 568 00:37:40,000 --> 00:37:46,000 way because there is an input that is asking it to move along. 569 00:37:45,000 --> 00:37:51,000 Now all I want to do is solve this in general with a Laplace 570 00:37:49,000 --> 00:37:55,000 transform. If I do it in general, 571 00:37:51,000 --> 00:37:57,000 that is always easier than doing it in particular since you 572 00:37:56,000 --> 00:38:02,000 don't ever have to do any calculations. 573 00:38:00,000 --> 00:38:06,000 It is s squared Y. There are no other terms here 574 00:38:05,000 --> 00:38:11,000 because the initial conditions are zero. 575 00:38:08,000 --> 00:38:14,000 This part will be a times s Y. 576 00:38:12,000 --> 00:38:18,000 Again, no other terms because the initial conditions are zero. 577 00:38:17,000 --> 00:38:23,000 Plus b times Y. And all that is equal to 578 00:38:21,000 --> 00:38:27,000 whatever the Laplace transform is of the right-hand side. 579 00:38:26,000 --> 00:38:32,000 So it is F of s. Next step. 580 00:38:31,000 --> 00:38:37,000 Boy, this is an easy problem. You solve for Y. 581 00:38:35,000 --> 00:38:41,000 Well, Y is F of s times one over s squared plus as plus b. 582 00:38:45,000 --> 00:38:51,000 Now, what is that? The next step now is to figure 583 00:38:50,000 --> 00:38:56,000 out what the answer to the problem is, what's the Y of t? 584 00:38:55,000 --> 00:39:01,000 Well, you do that by taking the 585 00:39:00,000 --> 00:39:06,000 inverse Laplace transform. But because these are general 586 00:39:04,000 --> 00:39:10,000 functions, I don't have to write down any specific answer. 587 00:39:09,000 --> 00:39:15,000 The only thing is to use the convolution because this is the 588 00:39:13,000 --> 00:39:19,000 product of two functions of s. The inverse transform will be 589 00:39:18,000 --> 00:39:24,000 the convolution of their respective things. 590 00:39:21,000 --> 00:39:27,000 The answer is going to be the convolution of F of t, 591 00:39:26,000 --> 00:39:32,000 the input function in other words, convoluted with the 592 00:39:30,000 --> 00:39:36,000 inverse Laplace transform of that thing. 593 00:39:35,000 --> 00:39:41,000 Now, we have to have a name for that, and those are the two 594 00:39:39,000 --> 00:39:45,000 words I want to introduce you to because they are used 595 00:39:43,000 --> 00:39:49,000 everywhere. The function, 596 00:39:44,000 --> 00:39:50,000 on the right-hand side, this function one over s 597 00:39:48,000 --> 00:39:54,000 squared plus as plus b, 598 00:39:51,000 --> 00:39:57,000 notice it only depends upon the left-hand side of the 599 00:39:55,000 --> 00:40:01,000 differential equation, on the damping constant. 600 00:40:00,000 --> 00:40:06,000 The spring constant if you are thinking of a mass spring 601 00:40:03,000 --> 00:40:09,000 dashpot system. So this depends only on the 602 00:40:06,000 --> 00:40:12,000 system, not on what input is going into it. 603 00:40:08,000 --> 00:40:14,000 And it is called the transfer function. 604 00:40:11,000 --> 00:40:17,000 Is usually called capital W of, sometimes it is 605 00:40:15,000 --> 00:40:21,000 capital H of s, there are different things, 606 00:40:18,000 --> 00:40:24,000 but it is always called the transfer function. 607 00:40:27,000 --> 00:40:33,000 What we are interested in putting here its inverse Laplace 608 00:40:31,000 --> 00:40:37,000 transform. Well, I will call that W of t 609 00:40:34,000 --> 00:40:40,000 to go with the capital W of s by the usual 610 00:40:39,000 --> 00:40:45,000 notation. Its inverse Laplace transform, 611 00:40:42,000 --> 00:40:48,000 well, I cannot calculate that. I will just give it a name, 612 00:40:46,000 --> 00:40:52,000 W of t. And that is called the weight 613 00:40:49,000 --> 00:40:55,000 function of the system. This is the transfer function 614 00:40:53,000 --> 00:40:59,000 of the system, so put in "of the system" if 615 00:40:57,000 --> 00:41:03,000 you are taking notes. And so the answer is that 616 00:41:02,000 --> 00:41:08,000 always the solution is the convolution to this differential 617 00:41:06,000 --> 00:41:12,000 equation that we have been solving for the last three or 618 00:41:11,000 --> 00:41:17,000 four weeks. It is the convolution of that. 619 00:41:14,000 --> 00:41:20,000 And, therefore, the solution is expressed as a 620 00:41:18,000 --> 00:41:24,000 definite integral of the function of the input on the 621 00:41:22,000 --> 00:41:28,000 right-hand side, what is forcing the equation, 622 00:41:26,000 --> 00:41:32,000 times this magic function but flipped and translated by t. 623 00:41:32,000 --> 00:41:38,000 That says du for you guys over there. 624 00:41:34,000 --> 00:41:40,000 In other words, the solution to the 625 00:41:37,000 --> 00:41:43,000 differential equation is presented as a definite 626 00:41:41,000 --> 00:41:47,000 integral. Marvelous. 627 00:41:42,000 --> 00:41:48,000 And the only thing is the definite integral involves this 628 00:41:47,000 --> 00:41:53,000 funny function W of t. To understand why that is the 629 00:41:52,000 --> 00:41:58,000 solution, you have to understand what W of t is. 630 00:41:55,000 --> 00:42:01,000 Well, formally, of course, it's that. 631 00:42:00,000 --> 00:42:06,000 But what does it really mean? The problem is what is W of t 632 00:42:05,000 --> 00:42:11,000 really? Not just formally, 633 00:42:08,000 --> 00:42:14,000 but what does it really mean? I mean, is it real? 634 00:42:12,000 --> 00:42:18,000 I think the simplest way of thinking of it, 635 00:42:16,000 --> 00:42:22,000 once you know about the delta function is just to think of 636 00:42:21,000 --> 00:42:27,000 this differential equation y double prime plus a y prime plus 637 00:42:27,000 --> 00:42:33,000 b. Except use as the input the 638 00:42:32,000 --> 00:42:38,000 Dirac delta function. In other words, 639 00:42:35,000 --> 00:42:41,000 we are kicking the mass. The mass starts at rest, 640 00:42:39,000 --> 00:42:45,000 so the initial conditions are going to be what they were 641 00:42:43,000 --> 00:42:49,000 before. y of zero, 642 00:42:46,000 --> 00:42:52,000 y prime of zero. Both zero. 643 00:42:49,000 --> 00:42:55,000 The mass starts at rest from the equilibrium position, 644 00:42:53,000 --> 00:42:59,000 and it is kicked in the positive direction, 645 00:42:57,000 --> 00:43:03,000 I guess that's this way, with unit impulse. 646 00:43:02,000 --> 00:43:08,000 At time zero with unit impulse. In other words, 647 00:43:05,000 --> 00:43:11,000 kick it just hard enough so you impart a unit impulse. 648 00:43:10,000 --> 00:43:16,000 So that situation is modeled by this differential equation. 649 00:43:15,000 --> 00:43:21,000 The kick at time zero is modeled by this input, 650 00:43:19,000 --> 00:43:25,000 the Dirac delta function. And now, what happens if I 651 00:43:23,000 --> 00:43:29,000 solve it? Well, you see, 652 00:43:25,000 --> 00:43:31,000 everything in the solution is the same. 653 00:43:30,000 --> 00:43:36,000 The left stays the same, but on the right-hand side I 654 00:43:34,000 --> 00:43:40,000 should have not f of s here. 655 00:43:37,000 --> 00:43:43,000 Since this is the delta function, I should have one. 656 00:43:42,000 --> 00:43:48,000 What I get is, on the left-hand side, 657 00:43:45,000 --> 00:43:51,000 s squared Y plus as Y plus bY equals, 658 00:43:50,000 --> 00:43:56,000 for the Laplace transform of the right-hand side is simply 659 00:43:56,000 --> 00:44:02,000 one. And, therefore, 660 00:43:57,000 --> 00:44:03,000 Y is what? Y is one over exactly the 661 00:44:02,000 --> 00:44:08,000 transform function. And therefore its inverse 662 00:44:05,000 --> 00:44:11,000 Laplace transform is that weight function. 663 00:44:09,000 --> 00:44:15,000 That is the simplest interpretation I know of what 664 00:44:13,000 --> 00:44:19,000 this magic weight function is, which gives the solution to all 665 00:44:18,000 --> 00:44:24,000 the differential equations, no matter what the input is. 666 00:44:23,000 --> 00:44:29,000 The weight function is the response of the system at rest 667 00:44:27,000 --> 00:44:33,000 to a sharp kick at time zero with unit impulse. 668 00:44:33,000 --> 00:44:39,000 And read the notes because they will explain to you why this 669 00:44:37,000 --> 00:44:43,000 could be thought of as the superposition of a lot of sharp 670 00:44:42,000 --> 00:44:48,000 kicks times zero a little later. Kick, kick, kick, 671 00:44:46,000 --> 00:44:52,000 kick. And that's what makes the 672 00:44:49,000 --> 00:44:55,000 solution. Next time we start systems.